You are on page 1of 36

FUNDAMENTALS OF CIRCUITS

28
Conceptual Questions
28.1. Calculate I = ΔV /R.

2V 1V
Ia = =1A Ib = = 0.5 A
2Ω 2Ω
2V 1V
Ic = =2A Id = =1A
1Ω 1Ω

Ic > Ia = Id > I b .

28.2. No. This is not a complete circuit. A connection from the outer metal case of the bulb to the negative end of
the battery is required to make a complete circuit.

28.3. ΔV12 = 3 V. There is no current through the resistor so there is no potential difference across the resistor.

28.4. R1 dissipates more power, since P = I 2 R and the same current flows through each resistor.

28.5. R2 dissipates more power, since P = (ΔV ) 2 /R and both resistors have the same potential drop ΔV because they
are connected in parallel.

28.6. Calculate P = ( ΔV )2 / R.
(ΔV )2
( 1 ΔV )
2
Pa = 1
R Pb = 2 = Pa
2R 8
(2ΔV ) 2 (2ΔV ) 2
Pc = 1R
= 8 Pa Pd = = 2 Pa
2
2R

Pc > Pd > Pa > Pb .

© Copyright 2017 Pearson Education, Inc. All rights reserved. This material is protected under all copyright laws as they currently exist.
No portion of this material may be reproduced, in any form or by any means, without permission in writing from the publisher.
28-1
28-2 Chapter 28

28.7. The two resistors are in series, as can be seen by redrawing the circuit as follows:

28.8. Increase. Recall ΔVbat = Rε /(R + r ), where ε is the ideal battery voltage. As R increases, ΔVbat also
increases. This makes sense since less current flows through the circuit, thus there is a smaller potential drop in the
battery due to its internal resistance. In the case of no current (battery disconnected, R → ∞), Δ Vbat = ε .

28.9. Bulb A gets brighter and bulb B goes out. When the switch is closed all the current travels along this zero
resistance path rather than through bulb B. With less resistance in the circuit the current is larger, so bulb A burns
brighter.

28.10. (a) A > B = C. All current flowing from the battery must go through bulb A, then the current splits at the
junction with half going through B and half through C. With the same resistance for each bulb, the larger current
means more brightness.
(b) Bulbs B and C go out because now there is a zero resistance wire along which all the current will flow. Now the
total resistance in the circuit has decreased, so the current increases and bulb A will burn brighter.

28.11. Applying Kirchhoff’s loop law around the outside edge of the circuit,
∑ Δ V1 = ε − IR − Δ V12 = 0 ⇒ Δ V12 = ε − IR

That is, the potential difference Δ V12 between points 1 and 2 is the potential supplied by the battery minus the
potential lost in resistor R. When bulb B is in place, a current I flows through the resistor and bulb. In that case, Δ V12
is less than ε . However, if bulb B is removed, the current no longer flows and I = 0 A. Thus, Δ V12 (no bulb) = ε .
Thus, the potential difference Δ V12 increases when bulb B is removed.

28.12. The brightness of each bulb stays the same. The ground wire is not part of a complete circuit so no current
will flow down it. The current through the bulbs remains the same.

28.13. R2 > R3 > R1. A larger resistance makes a larger time constant t = RC , which means it takes longer for the
voltage to decrease. The size of the initial voltage makes no difference. The R2 curve decays the most slowly, so R2
is the largest, and the R1 curve decays the most rapidly, so R1 is the smallest.

© Copyright 2017 Pearson Education, Inc. All rights reserved. This material is protected under all copyright laws as they currently exist.
No portion of this material may be reproduced, in any form or by any means, without permission in writing from the publisher.
Fundamentals of Circuits 28-3

Exercises and Problems

Exercises
Section 28.1 Circuit Elements and Diagrams
28.1. Solve:

From the circuit in Figure EX28.1, we see that the 50 and 100 Ω resistors are connected in series across the battery.
The 75 Ω resister is also connected across the battery in parallel with the first 50 and 100 Ω resistors.

28.2. Solve: In Figure EX28.2, the positive terminal of the battery is connected to the 50 Ω resistor, whose other
end is connected to the 100 Ω resistor and the capacitor, which are in parallel. Thus, we have a resistor connected in
series with a parallel combination of a resistor and a capacitor.

Section 28.2 Kirchhoff’s Laws and the Basic Circuit


28.3. Model: Assume that the connecting wires are ideal.
Visualize: Please refer to Figure EX28.3.
Solve: The current in the 2 Ω resistor is I1 = 4 V/2 Ω = 2 A to the right. The current in the 5 Ω resistor is
I 2 = (15 V)/(5 Ω) = 3 A downward. Let I be the current flowing out of the junction. Kirchhoff’s junction law then
gives
I = I1 − I 2 = 2 A − 3 A = − 1 A
Thus, 1 A of current flows into the junction (i.e., to the left).

28.4. Model: The batteries and the connecting wires are ideal.
Visualize: Please refer to Figure EX28.4.
Solve: (a) Choose the current I to be in the clockwise direction. If I ends up being a positive number, then the current
really does flow in this direction. If I is negative, the current really flows counterclockwise. There are no junctions,
so I is the same for all elements in the circuit. With the 9 V battery labeled 1 and the 6 V battery labeled 2,
Kirchhoff’s loop law gives
∑ Δ Vi = Δ Vbat 1 + Δ VR + ΔVbat 2 = +ε1 − IR − ε 2 = 0
ε1 − ε 2
9 V − 18 V
I= = = − 0.9 A
R 10 Ω
Note the signs: Potential is gained in battery 1, but potential is lost both in the resistor and in battery 2. Because I is
negative, we can say that I = 0.9 A and flows from right to left through the resistor.

© Copyright 2017 Pearson Education, Inc. All rights reserved. This material is protected under all copyright laws as they currently exist.
No portion of this material may be reproduced, in any form or by any means, without permission in writing from the publisher.
28-4 Chapter 28

(b) In the graph below, we start at the lower-left corner of the circuit and travel clockwise around the circuit (i.e.,
against the current). We start by losing 9 V going through battery 1, then loss Δ VR = − IR = 9 V going through the
resistor. We then gain 18 V going through battery 2. The final potential is the same as the initial potential, as required.

28.5. Model: Assume ideal connecting wires and an ideal battery for which Δ Vbat = ε .
Visualize: Please refer to Figure EX28.5. We will choose a clockwise direction for I. Note that the choice of the
current’s direction is arbitrary because, with two batteries, we may not be sure of the actual current direction. The 3 V
battery will be labeled 1 and the 6 V battery will be labeled 2.
Solve: (a) Kirchhoff’s loop law, going clockwise from the negative terminal of the 3 V battery, gives
Δ Vclosed loop = ∑ (ΔV )i = Δ Vbat 1 + Δ VR + Δ Vbat 2 = 0
i
9V
+3 V − (18 Ω) I + 6 V = 0 ⇒ I=
= 0.5 A
18 Ω
Thus, the current through the 18 Ω resistor is 0.5 A. Because I is positive, the current is to the right (i.e., clockwise).
(b)

Assess: The graph shows a 3 V gain in battery 1, a −9 V loss in the resistor, and a gain of 6 V in battery 2. The final
potential is the same as the initial potential, as required.

28.6. Model: Model the wires as ideal.


Visualize: Define the current I as a counterclockwise flow.
Solve: There are no junctions, so conservation of charge tells us that the same current flows through each circuit
element. From Kirchhoff’s loop law,
∑ Δ Vi = Δ Vbat + ΔV40 + Δ V60 = 0

© Copyright 2017 Pearson Education, Inc. All rights reserved. This material is protected under all copyright laws as they currently exist.
No portion of this material may be reproduced, in any form or by any means, without permission in writing from the publisher.
Fundamentals of Circuits 28-5

As we go around the circuit in the direction of the current (starting in the lower right corner), potential is gained in
the battery (ΔVbat = ε bat = + 50 V) and potential is lost in the resistors (ΔVres = − IR ). The loop law gives
ε bat 50 V 1
ε bat = − IR1 − IR2 = − I ( R12 + R33 ) ⇒ I = = = A
R40 + R60 100 Ω 2
Now that we know the current, we can find the potential difference across each resistor:
Δ V40 = IR40 = (1/2 A)(40 Ω) = 20 V
Δ V60 = IR60 = (1/2 A)(60 Ω) = 30 V
Assess: The sum of the potential differences across the resistors equals the potential across the battery, as it should.

Section 28.3 Energy and Power


28.7. Model: The 1500 W rating is for operating at 120 V.
Solve: The hair dryer dissipates 1500 W at Δ VR = 120 V. Thus, the hair dryer’s resistance is

(Δ VR ) 2 (120 V) 2
R= = = 9.60 Ω
PR 1500 W
The current in the hair dryer when it is used is given by Ohm’s law:
ΔVR 120 V
I= = = 12.5 A
R 9.60 Ω

28.8. Model: Assume ideal connecting wires and an ideal battery.


Visualize: Please refer to Figure EX28.8.
Solve: The power dissipated by each resistor can be calculated from Equation 28.12, PR = I 2 R, provided we can
find the current through the resistors. Let us choose a clockwise direction for the current and solve for the value of I
by using Kirchhoff’s loop law. Going clockwise from the negative terminal of the battery gives
∑ (ΔV )i = ΔVbat + ΔVR1 + ΔVR 2 = 0 ⇒ + 12 V − IR1 − IR2 = 0
i
12 V 12 V 2
I= = = A = 0.40 A
R1 + R2 12 Ω + 18 Ω 5
The power dissipated by resistors R1 and R2 is:

PR1 = I 2 R1 = (0.40 A) 2 (12 Ω) = 1.9 W PR2 = I 2 R2 = (0.40 A) 2 (18 Ω) = 2.9 W

28.9. Model: Assume ideal connecting wires and the resistors obey Ohm’s law.
Visualize: The current is the same in both bulbs. The stated power on each bulb is what it would dissipate if
connected singly across a 120 V source.
Solve: (a)
P60 (Δ V ) 2 / R60 (120 V) 2 /(60 W) 100 W
= = = >1
P100 (Δ V ) 2 / R100 (120 V)2 /(100 W) 60 W
Therefore, the 60 W bulb emits more power (or is brighter) than the 100 W bulb.
(b) The 60 W label means it dissipates 60 W when it “sees” 120 V (which it doesn’t in the circuit shown since the
bulbs are in series). This allows us to calculate the resistance of each bulb from the situation (not shown) where they
each see 120 V and each dissipates the power stated on the label.
( ΔV )2 (120 V)2 ( ΔV )2 (120 V)2
R60 = = = 240 Ω R100 = = = 144 Ω
P60 60 W P100 100 W
Now we compute the current in the circuit (the same at all places in the circuit). Use Ohm’s law.
(Δ V ) 120 V
I= = = 0.3125 A
Rtot 384 Ω

© Copyright 2017 Pearson Education, Inc. All rights reserved. This material is protected under all copyright laws as they currently exist.
No portion of this material may be reproduced, in any form or by any means, without permission in writing from the publisher.
28-6 Chapter 28

With the current we now compute the power dissipated by each bulb using P = I 2 R.
P60 = (0.3125 A) 2 (240 Ω) = 23 W P100 = (0.3125 A)2 (144 Ω) = 14 W
Assess: The bulbs do not dissipate the power stated on the label in this case because they do not each “see” 120 V
since they are in series. If the bulbs were in parallel across a 120 V source they would dissipate 60 W and 100 W
(and the 100 W bulb would be brighter).

28.10. Model: Assume ideal wires.


Solve: Because the bulbs are identical, they all have the same resistance, which we shall call R. The equivalent
resistance of the left-hand branch is
( Req ) L = R + (1/ R + 1/ R )−1 = R + R /2 = 3R /2
The equivalent resistance of the right-hand branch is
( Req ) R = R + R = 2 R
The voltage difference across the equivalent resistance of the left-hand and right-hand branches is ε , so the currents
flowing through the left- and right-hand branches are
ε 2ε ε 1ε
IL = = , IR = =
( Req )L 3R ( Req ) R 2 R

Thus, the current flowing through bulbs S and T is I R . The current flowing through bulb P is I L and that flowing
through bulbs Q and R is I L /2. Because the brightness is proportional to the power P = I 2 R, ordering the bulbs in
terms of current will give the same result as ordering them in terms of brightness. Thus, ordering the bulbs from
brightest to dimmest gives
P >S=T >Q=R
so the response is C.

28.11. Model: Assume ideal wires.


Visualize: The following equivalent circuits will be useful, where we have labeled three points in the circuits a, b,
and c. The unlabeled resistors all have resistance R.

Solve: Because the bulbs are identical, they all have the same resistance, which we shall call R. The bulb brightness
is proportional to the power P = V 2 / R, so we shall calculate the voltage difference across each bulb. First, we find
the potential at points a, b, and c with respect to point d (i.e., the negative terminal of the battery), which we shall
assign as the zero of the potential. The potential at point a is ε , so Va = ε . Considering the last equivalent circuit
above, we see that the current flowing around the circuit is
Va 5ε
I= =
R + 3R /5 8 R
Therefore, the potential at point b is
5ε 3ε
Vb = Va − IR = ε − =
8 8

© Copyright 2017 Pearson Education, Inc. All rights reserved. This material is protected under all copyright laws as they currently exist.
No portion of this material may be reproduced, in any form or by any means, without permission in writing from the publisher.
Fundamentals of Circuits 28-7

Considering the second-to-last circuit, we see that the current flowing from point b to point d must be
Vb 2 3ε ε
IR = = =
3R /2 3R 8 4 R
so, looking at the second circuit, we can find the potential at point c:
⎛ R ⎞ 3ε ⎛ ε ⎞⎛ R ⎞ ε
Vc = Vb − I R ⎜ ⎟ = −⎜ ⎟⎜ ⎟ =
⎝ 2 ⎠ 8 ⎝ 4 R ⎠⎝ 2 ⎠ 4
In the following table, we put the bulbs and the potential difference across them:

Bulb Potential Difference Power


5ε 25 ε 2
P Δ V = Vb − Va = −
8 64 R
3ε 9 ε2
Q Δ V = Vd − Vb = −
8 64 R
ε 1 ε2
R Δ V = Vc − Vb = −
8 64 R
ε 1 ε2
S Δ V = Vc − Vb = −
8 64 R
ε 4 ε2
T Δ V = Vd − Vc = −
4 64 R

Thus, ordering the bulbs from brightest to dimmest gives


P >Q >T >R =S
so the response is D.

28.12. Solve:
⎛ 3600 s ⎞
1 kWh = 1000 W ⋅ h = 1000 W ⋅ h ⎜ ⎟ = 3,600,000 W ⋅ s = 3,600,000 J
⎝ 1h ⎠
Assess: When you buy 1 kWh of energy from the “power company” for 10 cents you are getting a lot of joules!

28.13. Model: The 100 W rating is for operating at 120 V.


Solve: A standard bulb uses Δ V = 120 V. We can use the power dissipation to find the resistance of the filament:
ΔV 2 ΔV 2 (120 V) 2
P= ⇒ R= = = 144 Ω
R P 100 W
But the resistance is related to the filament’s geometry:
ρL ρL ρL (9.0 × 10−7 Ω m)(0.070 m)
R= = ⇒ r= = = 1.18 × 10−5 m = 11.8 μ m
A π r2 πR π (144 Ω)
The filament’s diameter is d = 2r = 24 μ m.

28.14. Solve: (a) The average power consumed by a typical American family is
kWh kWh 1000
Pavg = 1000 = 1000 = kW = 1.389 kW
month 30 × 24 h 720
Because P = (Δ V )I with ΔV being the voltage of the power line to the house,
Pavg 1389 W
I avg = = = 11.6 A
ΔV 120 V

© Copyright 2017 Pearson Education, Inc. All rights reserved. This material is protected under all copyright laws as they currently exist.
No portion of this material may be reproduced, in any form or by any means, without permission in writing from the publisher.
28-8 Chapter 28

(b) Because P = (Δ V ) 2 / R,
(Δ V ) 2 (120 V)2
Ravg = = = 10.4 Ω
Pavg 1389 W

28.15. Solve: The cost of running the waterbed 25% of the time for a year is
⎛ 24 hr ⎞ ⎛ kW ⎞⎛ $0.12 ⎞
(0.25)(450 W)(365 days) ⎜ ⎟⎜ ⎟⎜ ⎟ = $118
⎝ day ⎠ ⎝ 1000 W ⎠⎝ kW hr ⎠

To two significant figures, the cost is $120.

Section 28.4 Series Resistors


Section 28.5 Real Batteries
28.16. Model: Assume ideal connecting wires and an ideal battery.
Solve: As shown in Figure EX28.16, a potential difference of 5.0 V causes a current of 100 mA through the three
resistors in series. The situation is the same if we replace the three resistors with an equivalent resistor Req . That is, a
potential difference of 5.0 V across Req causes a current of 100 mA through it. From Ohm’s law,
ΔVR 5.0 V
Req = ⇒ R + 15 Ω + 10 Ω = ⇒ R = 25 Ω
I 100 mA

28.17. Visualize: Please refer to Figure EX28.17.


Solve: (a) Apply Kirchhoff’s circuit law to the circuit shown in Figure EX28.17:
ε 1.5 V
Δ Vbatt = 0 ⇒ ε − Ir = 0 ⇒ r = = = 0.65 Ω
I 2.3 A
(b) The power dissipated is
P = I 2r = (2.3 A) 2 (0.65 Ω) = 3.5 W

28.18. Model: The voltage source and the connecting wires are ideal.
Visualize: Please refer to Figure EX28.18.
Solve: Let us first apply Kirchhoff’s loop law starting clockwise from the lower-left corner:
Vin
+Vin − IR − I (100 Ω) = 0 V ⇒ I=
R + 100 Ω
The output voltage is
⎛ Vin ⎞ Vout 100 Ω
Vout = (100 Ω) I = (100 Ω) ⎜ ⎟ ⇒ V = R + 100 Ω
⎝ R + 100 Ω ⎠ in

For Vout = Vin /10, the above equation can be simplified to obtain R:
Vin /10 100 Ω
= ⇒ R + 100 Ω = 1000 Ω ⇒ R = 900 Ω
Vin R + 100 Ω

28.19. Model: Assume ideal connecting wires but not an ideal battery.
Visualize: Please refer to Figure 28.18.
Solve: From Equation 28.19, the potential difference across the battery is
R ⎛ ε ⎞ ⎛ 9.0 V ⎞
Δ Vbat = ⇒ r = R⎜ − 1⎟ = (20 Ω) ⎜ − 1⎟ = 1.2 Ω
R+r ⎝ ΔVbat ⎠ ⎝ 8.5 V ⎠
Assess: 1 Ω is a typical internal resistance for a battery. This causes the battery’s terminal voltage in the circuit to
be 0.5 V less than its emf.

© Copyright 2017 Pearson Education, Inc. All rights reserved. This material is protected under all copyright laws as they currently exist.
No portion of this material may be reproduced, in any form or by any means, without permission in writing from the publisher.
Fundamentals of Circuits 28-9

28.20. Model: Assume ideal connecting wires but not an ideal battery.
Visualize: The circuit for an ideal battery is the same as the circuit in Figure EX28.20, except that the 1 Ω resistor
is not present.
Solve: In the case of an ideal battery, we have a battery with ε = 15 V connected to two series resistors of 10 Ω
and 20 Ω resistance. Because the equivalent resistance is Req = 10 Ω + 20 Ω = 30 Ω and the potential difference
across Req is 15 V, the current in the circuit is
ΔV ε 15 V
I= = = = 0.50 A
Req Req 30 Ω
The potential difference across the 20 Ω resistor is
Δ V20 = IR = (0.50 A)(20 Ω) = 10 V
In the case of a real battery, we have a battery with ε = 15 V connected to three series resistors: 10 Ω, 20 Ω, and an
internal resistance of 1 Ω. Now the equivalent resistance is
′ = 10 Ω + 20 Ω + 1 Ω = 31 Ω
Req
The potential difference across Req is the same as before ε = 15 V. Thus,
ΔV ′ ε 15 V
I′ = = = = 0.4839 A

Req ′
Req 31 Ω
Therefore, the potential difference across the 20 Ω resistor is
′ = I ′R = (0.4839 A)(20 Ω) = 9.68 V
Δ V20
That is, the potential difference across the 20 Ω resistor is reduced from 10 V to 9.68 V due to the internal
resistance of 1 Ω of the battery. The percentage change in the potential difference is
⎛ 10.0 V − 9.68 V ⎞
⎜ 10.0 V ⎟ × 100% = 3.2%
⎝ ⎠

28.21. Model: Assume that the connecting wires are ideal but the battery is not ideal.
Visualize:

Solve: The figure shows a variable resistor R connected across the terminals of a battery that has an emf ε and an
internal resistance r. Using Kirchhoff’s loop law and starting from the lower-left corner gives
+ε − Ir − IR = 0 ⇒ ε = I (r + R )
From the point in Figure EX28.21 that corresponds to R = 0 Ω, we have
ε = (6 A)(r + 0 Ω) = (6 A)r
From the point that corresponds to R = 10 Ω, we have
ε = (3 A)(r + 10 Ω)
Combining the two equations gives
(6 A)r = (3 A)(r + 10 Ω) ⇒ 2r = r + 10 Ω ⇒ r = 10 Ω
Also, ε = (3 A)(10 Ω + 10 Ω) = 60 V.

© Copyright 2017 Pearson Education, Inc. All rights reserved. This material is protected under all copyright laws as they currently exist.
No portion of this material may be reproduced, in any form or by any means, without permission in writing from the publisher.
28-10 Chapter 28

Assess: With ε = 60 V and r = 10 Ω, the equation ε = I (r + R) is satisfied by all values of R and I on the graph in
Figure EX28.21.

Section 28.6 Parallel Resistors


28.22. Visualize: The three resistors in Figure EX28.22 are equivalent to a resistor of resistance Req = 75 Ω.
Solve: Because the three resistors are in parallel,
1 1 1 1 2 1 400 Ω + R (200 Ω) R 200 Ω
= + + = + = ⇒ Req = 75 Ω = =
Req R 200 Ω R R 200 Ω (200 Ω) R (400 Ω + R) ⎛ 400 Ω ⎞
1+ ⎜ ⎟
⎝ R ⎠
400 Ω
R= = 240 Ω
200 Ω
−1
75 Ω

28.23. Model: Assume ideal connecting wires.


Visualize: Please refer to Figure EX28.23.
Solve: The resistance R is given by Ohm’s law, R = Δ VR / I R . To determine I R we use Kirchhoff’s junction law.
The input current I splits into the three currents I10 , I15 , and I R . That is,
8V 8V 20 2
2.0 A = I10 + I15 + I R = + + IR ⇒ I R = 2.0 A − 15 A= 3
A
10 Ω 15 Ω
Using this value of I R in Ohm’s law gives
8V
R= 2 A
= 12 Ω
3

28.24. Visualize:

The figure shows a metal wire of resistance R that is cut into two pieces of equal length. This produces two wires
each of resistance R/2.
Solve: Since these two wires are connected in parallel,
1 1 1 2 2 4 R
= + = + = ⇒ Req =
Req R /2 R /2 R R R 4

28.25. Model: The connecting wires are ideal with zero resistance.
Solve:

© Copyright 2017 Pearson Education, Inc. All rights reserved. This material is protected under all copyright laws as they currently exist.
No portion of this material may be reproduced, in any form or by any means, without permission in writing from the publisher.
Fundamentals of Circuits 28-11

For the first step, the two 60 W resistors and the 40 Ω resistor are in parallel. Their equivalent resistance is
1 1 1 1
= + = ⇒ Req 1 = 24 Ω
Req 1 60 Ω 40 Ω 24 Ω
For the second step, the two 60 W resistors and the 45 Ω resistor are in parallel. Their equivalent resistance is
1 2 1 5
= + = ⇒ Req 2 = 18 Ω
Req 2 60 Ω 45 Ω 90 Ω

For the third step, resistors Req 1 = 24 Ω and Req 2 = 18 Ω are in series. Therefore,
Req 3 = Req 1 + Req 2 = 24 Ω + 18 Ω = 42 Ω
For the fourth step, the resistors 21 Ω and Req 3 = 42 Ω are in parallel. So,
1 1 1
= + ⇒ Req 4 = 14 Ω
Req 4 21 Ω 42 Ω
The equivalent resistance of the circuit is 14 Ω.

28.26. Model: The connecting wires are ideal with zero resistance.
Solve:

For the first step, the 10 Ω and 40 Ω resistors are in series and the equivalent resistance is 50 Ω. Likewise, the
55 Ω and 20 Ω resistors give an equivalent resistance of 75 Ω. For the second step, the 75 Ω and 50 Ω resistors
are in parallel and the equivalent resistance is
−1
⎡ 1 1 ⎤
⎢ 50 Ω + 75 Ω ⎥ = 30 Ω
⎣ ⎦
For the third step, the 30 Ω and 10 Ω resistors are in series and the equivalent resistance is 40 Ω.

28.27. Model: The connecting wires are ideal with zero resistance.
Solve:

© Copyright 2017 Pearson Education, Inc. All rights reserved. This material is protected under all copyright laws as they currently exist.
No portion of this material may be reproduced, in any form or by any means, without permission in writing from the publisher.
28-12 Chapter 28

For the first step, the two resistors in the middle of the circuit are in parallel, so their equivalent resistance is
1 1 1
= + ⇒ Req 1 = 50 Ω
Req 1 100 Ω 100 Ω
The three 100 Ω resistors at the end are in parallel. Their equivalent resistance is
1 1 1 1
= + + ⇒ Req 2 = 33.3 Ω
Req 2 100 Ω 100 Ω 100 Ω
For the second step, the three resistors are in series, so their equivalent resistance is
100 Ω + 50 Ω + 33.3 Ω = 183 Ω
The equivalent resistance of the circuit is 183 Ω.

28.28. Model: The connecting wires are ideal with zero resistance.
Solve:

In the first step, the resistors 100 Ω, 100 Ω, and 100 Ω in the top branch are in series. Their combined resistance is
300 Ω. In the middle branch, the two resistors, each 100 Ω, are in series, so their equivalent resistance is 200 Ω. In
the second step, the three resistors are in parallel. Their equivalent resistance is
1 1 1 1
= + + ⇒ Req = 54.5 Ω
Req 300 Ω 200 Ω 100 Ω
The equivalent resistance of the circuit is 54.5 Ω.

28.29. Model: The connecting wires are ideal, but the battery is not.
Visualize: Please refer to Fig. EX28.29. We will designate the current in the 5 Ω resistor I 5 and the voltage drop
Δ V5 . Similar designations will be used for the other resistors.
Solve: Since the 10 Ω resistor is dissipating 40 W,
2 P10 40 W
P10 = I10 R10 = 40 W ⇒ I10 = = = 2.0 A
R10 10 Ω
Because the 5 Ω resistor is in series with the 10 Ω resistor, the same current must run through the 5 Ω resistor.
Therefore, the power dissipated by the 5 Ω resistor is
P5 = I 2 R5 = (2.0 A) 2 (5 Ω) = 20 W
The potential drop across the two left-hand resistors is the same as that across the right-hand resistor, so the power
dissipated by the 20 Ω resistor is
2
V20 (V + V ) 2 (I R + I R ) 2 (2.0 A) 2 (5 Ω + 10 Ω)2
P20 = = 5 10 = 10 5 10 10 = = 45 W
R20 R20 R20 20

© Copyright 2017 Pearson Education, Inc. All rights reserved. This material is protected under all copyright laws as they currently exist.
No portion of this material may be reproduced, in any form or by any means, without permission in writing from the publisher.
Fundamentals of Circuits 28-13

Section 28.8 Getting Grounded


28.30. Model: The connecting wires and the batteries are ideal with zero resistance.
Solve: By definition, the potential at the grounded point is zero. The 9 V battery raises the potential of point a 9 V
above ground, so point a is at 9 V. Now apply Kirchhoff’s current law to the circuit to find the current running
through it. Starting at the lower-left (grounded) corner and proceeding clockwise, this gives
−3 V
9 V − I (2 Ω) − 6 V − I (1 Ω) = 0 ⇒ I = =1 A
−3 Ω
Thus, the potential difference across the 2 Ω resistor is Δ Vres = −(1 A)(2 Ω) = − 2 V, so the potential at the upper-
right corner is 9 V − 2 V = 7 V. The potential drop going through the right-side battery is 6 V, so point b is 7 V −
6 V = 1 V. Thus, the potential at point a is at 9 V and the potential at point b is at 1 V.
Assess: To check, we can find the potential difference across the bottom resistor and verify that we find a potential
of zero volts at the grounded point. The potential difference across the bottom resistor is
Δ Vres = −(1 A)(1 Ω) = − 1 V, which when added to the potential at point b gives zero volts, as expected.

28.31. Model: The connecting wires and the batteries are ideal with zero resistance.
Solve: By definition, the potential at the grounded point is zero. The 5 V battery raises the potential of point a 5 V
above ground, so point a is at 5 V. Now apply Kirchhoff’s current law to the circuit to find the current running
through it. Starting at the lower-right (grounded) corner and proceeding counterclockwise, this gives
10 V
5 V − I (4 Ω) − 15 V − I (1 Ω) = 0 ⇒ I= = −2 A
−5 Ω
(Note that the negative sign means the current actually flows clockwise.) Thus, the potential difference across the 2 Ω
resistor is Δ Vres = −( −2 A)(4 Ω) = 8 V, so the potential at the upper-left corner is 5 V + 8 V = 13 V. The potential
drop going through the left-side battery is 15 V, so point b is 13 V − 15 V = − 2 V. Thus, the potential at point a is at
5 V and the potential at point b is at − 2 V
Assess: To check, we can find the potential difference across the bottom resistor and verify that we find a potential of
zero volts at the grounded point. The potential difference across the bottom resistor is Δ Vres = −(− 2 A)(1 Ω) = 2 V,
which when added to the potential at point b gives zero volts, as expected.

Section 28.9 RC Circuits


28.32. Solve: Noting that the unit of resistance is the ohm (V/A) and the unit of capacitance is the farad (C/V), the
unit of RC is
V C C C
RC = × = = =s
A V A C/s

28.33. Model: Assume ideal wires as the capacitors discharge through the two 1 k Ω resistors.
Visualize: The circuit in the figure has an equivalent circuit with resistance Req and capacitance Ceq .
Solve: The equivalent capacitance is Ceq = 2 μ F + 2 μ F = 4 μ F, and the equivalent resistance is
Req = 1 k Ω + 1 k Ω = 2 k Ω
Thus, the time constant for the discharge of the capacitors is
τ = ReqCeq = (2 kΩ)(4 μ F) = 8 × 10−3 s = 8 ms

28.34. Model: Assume ideal wires as the capacitors discharge through the two 1 kΩ resistors.
Visualize: The circuit in the figure has an equivalent circuit with resistance Req and capacitance Ceq .

© Copyright 2017 Pearson Education, Inc. All rights reserved. This material is protected under all copyright laws as they currently exist.
No portion of this material may be reproduced, in any form or by any means, without permission in writing from the publisher.
28-14 Chapter 28

Solve: The equivalent capacitance is


1 1 1
= + ⇒ Ceq = 1 μ F
Ceq 2 μ F 2 μ F
and the equivalent resistance is
1 1 1
= + ⇒ Req = 0.5 k Ω
Req 1 k Ω 1 k Ω
Thus, the time constant for the discharge of the capacitors is
τ = ReqCeq = (0.5 k Ω)(1 μ F) = 0.5 × 10−3 s = 0.5 ms
Assess: The discharge of the capacitors could easily be seen on an oscilloscope.

28.35. Model: The capacitor discharges through a resistor. Assume that the wires are ideal.
Solve: The decay of the capacitor charge is given by the Equation 28.31: Q = Q0e −t /τ . The time constant is
τ = RC = (1.0 k Ω)(10 μ F) = 0.010 s
The initial charge on the capacitor is Q0 = 20 μ C and it decays to 10 μC in time t. That is,

⎛ 10 μ C ⎞ τ
10 μ C = (20 μ C)e −τ /(0.010 s) ⇒ ln ⎜ ⎟=− ⇒ − τ = (0.010 s)(ln 2) = 6.9 ms
⎝ 20 μ C ⎠ 0.010 s

28.36. Model: The capacitor discharges through a resistor. Assume ideal wires.
Visualize: The switch in the circuit in Figure EX28.36 is in position a. When the switch is in position b the circuit
consists of a capacitor and a resistor.
Solve: (a) The switch has been in position a for a long time, which means the capacitor is fully charged to a charge
Q0 = C ΔV = Cε = (4 μ F)(9 V) = 36 μ C
Immediately after the switch is moved to the b position, the charge on the capacitor is Q0 = 36 μ C. The current
through the resistor is
Δ VR 9V
I0 = = = 0.36 A
R 25 Ω
Note that, as soon as the switch is closed, the potential difference across the capacitor Δ VC appears across the 25 Ω resistor.
(b) The charge Q0 decays as Q = Q0e −t /τ , where
τ = RC = (25 Ω)(4 μ F) = 100 μs
Thus, at t = 50 μs, the charge is

Q = (36 μ C)e −50 μs/100 μs = (36 μ C)e−0.5 = 22 μ C


and the resistor current is
I = I 0e−t /τ = (0.36 A)e−50 μs/100 μ s = 0.22 A
(c) Likewise, at t = 200 μs, the charge is Q = 4.9 μ C and the current is I = 49 mA.

28.37. Model: A capacitor discharges through a resistor. Assume ideal wires.


Solve: A capacitor initially charged to Q0 decays as Q = Q0e −t /RC . We wish to find R so that a 1.0 μ F capacitor
will discharge to 10% of its initial value in 2.0 ms. That is,
2.0 × 10−3 s −2.0 × 10−3 s
(0.10)Q0 = Q0e − (2.0 ms)/R (1.0 μ F) ⇒ ln(0.10) = − −6
⇒ R= = 0.87 k Ω
R(1.0 × 10 F) (1.0 × 10−6 F)ln(0.10)

Assess: A time constant of τ = RC = (870 Ω)(1.0 × 10−6 F) = 0.87 ms is reasonable.

© Copyright 2017 Pearson Education, Inc. All rights reserved. This material is protected under all copyright laws as they currently exist.
No portion of this material may be reproduced, in any form or by any means, without permission in writing from the publisher.
Fundamentals of Circuits 28-15

28.38. Model: A capacitor discharges through a resistor. Assume ideal wires.


Solve: The discharge current or the resistor current follows Equation 28.33: I = I 0e −t /RC . We wish to find the
capacitance C so that the resistor current will decrease to 25% of its initial value in 2.5 ms. That is,
2.5 × 10−3 s
0.25 I 0 = I 0e − (2.5 ms)/(100 Ω )C ⇒ ln(0 . 25) = − ⇒ C = 18 μ F
(100 Ω)C

Problems

28.39. Solve: Let the units guide you. For the incandescent bulb, the life-cycle cost pbulb is
pbulb = $0 .50 + (0 . 060 kW)(0 . 10 $/kW h)(10 ,000 h) = $60 . 50
This will give 1,000 hours, so the cost for 10,000 hours is $65.00. For the fluorescent tube, the cost for 10,000 hours is
ptube = $5 + (0.015 kW)(0.10 $/kW h)(10,000 h) = $20
Assess: The lifetime cost of the fluorescent bulb is one-third that of the incandescent bulb.

28.40. Solve: (a) The cost per month of the 1000 W refrigerator is
⎛ 1 kW ⎞⎛ 30 day ⎞ ⎛ 24 h ⎞ ⎛ $0.10 ⎞
(1000 W) ⎜ ⎟⎜ ⎟ ⎜ ⎟ (0.20) ⎜ ⎟ (1 month) = $14.40
⎝ 1000 W ⎠⎝ 1 month ⎠ ⎝ 1 day ⎠ ⎝ 1 kW h ⎠

28.41. Model: Assume ideal connecting wires and an ideal power supply.
Visualize:
.

The two light bulbs are basically two resistors in series.


Solve: A 75 W (120 V) light bulb has a resistance of
Δ V 2 (120 V)2
R= = = 192 Ω
P 75 W
The combined resistance of the two bulbs is
Req = R1 + R2 = 192 Ω + 192 Ω = 384 Ω
The current I flowing through Req is
Δ V 120 V
I= = = 0.3125 A
Req 384 Ω

Because Req is a series combination of the two equivalent resistors, the current 0.3125 A flows through both
resistors. Thus the power dissipated by each bulb is
P = I 2 R1 = (0.3125 A) 2 (192 Ω) = 19 W

28.42. Model: Assume the eel is an ideal battery and that it connects to its prey by ideal connecting wires.
Solve: (a) The power delivered by the eel is P = VI . The energy of the pulse is E = Pt , or
E = VIt = (450 V)(0.80 A)(1.0 ms) = 0.36 J
(b) The total charge that flows is Q = It = (0.80 A)(1.0 ms) = 0.80 mC.

© Copyright 2017 Pearson Education, Inc. All rights reserved. This material is protected under all copyright laws as they currently exist.
No portion of this material may be reproduced, in any form or by any means, without permission in writing from the publisher.
28-16 Chapter 28

28.43. Model: The wires and battery are ideal.


Visualize:

Solve: We can find the equivalent resistance necessary for the battery to deliver 9 W of power:
(ΔV )2 ( ΔV )2 (6.0 V)2
P= ⇒ R= = = 4.0 Ω
R P 9.0 W
The combination of the 2.0 Ω, 3.0 Ω, and 6.0 Ω resistors that make 4.0 Ω is shown in the figure above. The
3.0 Ω and 6.0 Ω parallel combination has an equivalent resistance of 2.0 Ω, which when added to the 2.0 Ω
resistor in series totals 4.0 Ω equivalent resistance.

28.44. Model: The battery is ideal.


Solve: The variable-resistance wire may be thought of as an infinite series of infinitesimally small resistors of
resistance
ρ ( x)dx
dR =
A
where A is the cross-sectional area of the wire. Because resistors in series add, we can integrate this expression to
find the total resistance R of the wire:
2.0 m 2.0 m ⎧ ⎡ ⎛ x ⎞2 ⎤ ⎫
1 1 ⎪ −6 ⎢ ⎥ ⎪
R=
A ∫ ρ (x)dx =
A ∫ ⎨ (2.5 × 10 ) 1 + ⎜ ⎟
⎢ ⎝ 1.0 m ⎠ ⎥
Ω m ⎬ dx
0 0 ⎪⎩ ⎣ ⎦ ⎪⎭
2.0 m
2.5 × 10−6⎛ x3 ⎞ 2.5 × 10−6 ⎛ (2.0 m)3 ⎞
= ⎜x+
2⎜
⎟ = ⎜ 2.0 m + ⎟ = 14.85 Ω
−3
π (0.50 × 10 m) ⎝ 3.0 m 2 ⎟⎠0 π (0.50 × 10 −3
m) ⎜⎝
2
3.0 m 2 ⎟⎠

We can use Ohm’s law to find the current through this wire if it were attached to a 9.0 V battery:
V 9.0 V
I= = = 0.61 A
R 14.85 Ω

28.45. Model: Use the laws of series and parallel resistances.


Visualize:

© Copyright 2017 Pearson Education, Inc. All rights reserved. This material is protected under all copyright laws as they currently exist.
No portion of this material may be reproduced, in any form or by any means, without permission in writing from the publisher.
Fundamentals of Circuits 28-17

Solve: Despite the diagonal orientation of the 12 Ω resistor, the 6 Ω, 12 Ω, and 4 Ω resistors are in parallel
because they have a common connection at both the top end and at the bottom end. Their equivalent resistance is
−1
⎛ 1 1 1 ⎞
Req = ⎜ + + ⎟ =2 Ω
⎝ 6 Ω 12 Ω 4 Ω ⎠
The trickiest issue is the 10 Ω resistor. It is in parallel with a wire, which is the same thing as a resistor with R = 0 Ω.
The equivalent resistance of 10 Ω in parallel with 0 Ω is
−1
⎛ 1 1 ⎞ 1
Req = ⎜ + ⎟ = (∞ ) − 1 = = 0 Ω
⎝ 10 Ω 0 Ω ⎠ ∞
In other words, the wire is a short circuit around the 10 Ω, so all the current goes through the wire rather than the
resistor. The 10 Ω resistor contributes nothing to the circuit. So the total circuit is equivalent to a 2 Ω resistor in
series with the 2 Ω equivalent resistance in series with the final 3 Ω resistor. The equivalent resistance of these
three series resistors is
Rab = 2 Ω + 2 Ω + 3 Ω = 7 Ω

28.46. Model: Assume that the connecting wires are ideal, but the battery is not. The battery has internal
resistance. Also assume that the ammeter does not have any resistance.
Visualize: Please refer to Figure P28.46.
Solve: When the switch is open,
ε − Ir − I (5.0 Ω) = 0 ⇒ V = (1.636 A)( r + 5.0 Ω)
where we applied Kirchhoff’s loop law starting from the lower-left corner. When the switch is closed, the current I
comes out of the battery and splits at the junction. The current I ′ = 1.565 A flows through the 5.0 Ω resistor and the
rest (I − I ′) flows through the 10.0 Ω resistor. Because the potential differences across the two resistors are equal,
I ′(5.0 Ω) = ( I − I ′)(10.0 Ω) ⇒ (1.565 A)(5.0 Ω) = ( I − 1.565 A)(10.0 Ω) ⇒ I = 2.348 A
Applying Kirchhoff’s loop law to the left loop of the closed circuit gives
ε − Ir − I ′(5.0 Ω) = 0 V ⇒ ε = (2.348 A) r + (1.565 A)(5.0 Ω) = (2.348 A)r + 7.825 V
Combining this equation for ε with the equation obtained from the circuit when the switch was open gives
(2.348 A)r + 7.825 V = (1.636 A)r + 8.18 V ⇒ (0.712 A)r = 0.355 V ⇒ r = 0.50 Ω
We also have ε = (1.636 A)(0.50 Ω + 5.0 Ω) = 9.0 V.

28.47. Model: Model the wires as ideal, including the fuse of the burned out bulb (no resistance).
Visualize: Call the resistance of one working bulb R. The voltage drop across one bulb when there are 50 working
bulbs is 120 V/50 and when there are only 49 working bulbs the voltage drop across one bulb is 120 V/49 (which
is greater).
Solve: Use P = (Δ V ) 2 / R to calculate the power dissipated in 1 out of 49 bulbs divided by the power dissipated by 1
out of 50 bulbs.
P1/ 49 (120 V/49) 2 / R 502
= = = 1.04
P1/50 (120 V/50) 2 / R 492
This is an increase of 4%.
Assess: Thus, the more bulbs that burn out, the more likely it is that yet another one will burn out. By the way, when
the first bulb burns out the power dissipated by the whole string only increases by 2% rather than 4% because there
are fewer bulbs.

28.48. Model: Model the helium as an ideal gas. The balloon is expandable.
Visualize: This is an integrated problem that we’ll do in various steps. First figure out the power dissipated by the
circuit. That energy will heat the gas and cause it to expand. We are given P = 1.2 atm = 121,560 Pa.

© Copyright 2017 Pearson Education, Inc. All rights reserved. This material is protected under all copyright laws as they currently exist.
No portion of this material may be reproduced, in any form or by any means, without permission in writing from the publisher.
28-18 Chapter 28

Solve: First compute the power dissipated by the resistors in the circuit. The equivalent resistance of the circuit is
−1
⎛ 1 1 ⎞
Req = 280 Ω + ⎜ + ⎟ = 400 Ω
⎝ 200 Ω 300 Ω ⎠
So the power dissipated by all the resistors together is
(Δ V ) 2 (45 V) 2
Pdiss = = = 5.06 W
Req 400 Ω
Now change our focus to the helium gas. Its energy changes as the temperature changes. For a monatomic gas like
helium,
Eth = 32 nRT ⇒ ΔEth = 32 nRΔT

But from the ideal gas law with the pressure held constant
PV = nRT ⇒ PΔV = nRΔT
where V here is volume whereas it was potential in the equation earlier. Combine the energy equation with the ideal
gas law:
ΔEth = 32 nRΔT = 32 PΔV

Now we compute the change in volume of the balloon as it heats up.


ΔV = 43 π (16/2 cm)3 − 43 π (15/2 cm)3 = 43 π [(8.0 cm)3 − (7.5 cm)3 ] = 3.78 × 10−4 m3

Finally we can compute the time required.


3
ΔEth 2 PΔV
3 (121,560
2
Pa)(3.78 × 10−4 m3 )
Δt = = = = 14 s
Pdiss Pdiss 5.06 W
Assess: This seems like a reasonable amount of time for the balloon to expand that much.

28.49. Model: Model the connecting wires as ideal.


Visualize: The equivalent resistance is much higher when they are in series than when they are in parallel, so we
expect the power dissipated to be greater when they are in parallel.
Solve: The equivalent resistance of the parallel circuit is
−1
⎛ 1 1 1 ⎞
Req = ⎜ + + ⎟ = 1.1 k Ω
⎝ 2.5 k Ω 3.5 k Ω 4.5 k Ω ⎠
So the power delivered to the resistors in parallel is
(Δ V ) 2 (100 V) 2
Pparallel = = = 9.1 W T
Req 1.1 k Ω
The equivalent resistance of the series circuit is
Req = 2.5 kΩ + 3.5 k Ω + 4.5 k Ω = 10.5 k Ω
So the power delivered to the resistors in series is
(Δ V ) 2 (100 V)2
Pseries = = = 0.95 W
Req 10.5 k Ω
The ratio is
Pparallel 9.1 W
= = 9.5
Pseries 0.95 W
Assess: The power dissipated by the parallel arrangement is indeed greater, as we expected.

© Copyright 2017 Pearson Education, Inc. All rights reserved. This material is protected under all copyright laws as they currently exist.
No portion of this material may be reproduced, in any form or by any means, without permission in writing from the publisher.
Fundamentals of Circuits 28-19

28.50. Model: Model the connecting wires as ideal.


Visualize: The description of the circuit is unambiguous, so we interpret the hint not to mean totally different
arrangements of circuit elements, but that there might be different bulbs (with different resistances) that might each
dissipate 10 W of power in the circuit shown. This leads us to look for a quadratic equation in Rb.

Solve: First use the loop law. The current is the same at all points around the circuit, so I does not need a subscript.
ε
ε − IRb − IR2 = 0 ⇒ I =
Rb + R2
Insert this value of I into the equation for power dissipated in the bulb. This gives a quadratic equation in Rb.
2
2 ⎛ ε ⎞ 2 2 2 2 2
Pb = I Rb = ⎜ ⎟ Rb ⇒ Pb ( Rb + R2 ) = ε Rb ⇒ Pb Rb + (2 Pb R2 − ε ) Rb + Pb R2 = 0
⎝ Rb + R2 ⎠
Use the quadratic formula.
−(2 Pb R2 − ε 2 ) ± (2 Pb R2 − ε 2 ) 2 − 4 Pb2 Rb2
Rb =
2 Pb

−(2(10 W)(2 Ω) − (9.0 V) 2 ) ± (2(10 W)(2 Ω) − (9.0 V) 2 ) 2 − 4(10 W) 2 (2.0 Ω) 2


=
2(10 W)
41 ± 9 5 8
= Ω = Ω or Ω
20 2 5
For each value of Rb we find the current.
9.0 V 9.0 V
I 5/ 2 = = 2.0 A I8/5 = = 2.5 A
2.5 Ω + 2.0 Ω 1.6 Ω + 2.0 Ω
Assess: These are typical currents.

28.51. Model: The connecting wires are ideal.


Visualize: Please refer to Figure 28.20.
Solve: (a) The internal resistance r and the load resistance R are in series, so the total resistance is R + r and the
current flowing in the circuit due to the emf ε is I = ε /(R + r ). The power dissipated by the load resistance is

ε 2R
P = I 2R =
( R + r )2
This is not the power ε I generated by the battery, but simply the power dissipated by the load R. The power is a
function of R, so we can find the maximum power by setting dP /dR = 0:
dP ε2 2ε 2 R ε 2 ( R + r ) − 2ε 2 R ε 2 (r − R )
= − = = =0
dR ( R + r ) 2 ( R + r )3 ( R + r )3 ( R + r )3
That is, the power dissipated by R is a maximum when R = r.

© Copyright 2017 Pearson Education, Inc. All rights reserved. This material is protected under all copyright laws as they currently exist.
No portion of this material may be reproduced, in any form or by any means, without permission in writing from the publisher.
28-20 Chapter 28

(b) The load’s maximum power dissipation will occur when R = r = 1.0 Ω, in which case
ε 2R (9.0 V) 2 (1.0 Ω)
P= 2
= = 20 W
(R + r) (2.0 Ω) 2
(c) When R is very small (R → 0 Ω), the current is a maximum (I → ε / r ) but the potential difference across the
load is very small (Δ V = IR → 0 V). So the power dissipation of the load is also very small (P = I Δ V → 0 W).
When R is very large (R → ∞), the potential difference across the load is a maximum (Δ V → ε ) but the current is
very small (I → 0 A). Once again, P is very small. If P is zero both for R → 0 and for R → ∞, there must be some
intermediate value of R where P is a maximum.

28.52. Model: The batteries are ideal, the connecting wires are ideal, and the ammeter has a negligibly small
resistance.
Visualize: Use the loop law and the junction law.
Solve: We can determine I1 by applying Kirchhoff’s loop law to the top loop. Starting clockwise from the upper-
left corner,
+ 9.0 V − I1 (3.0 Ω) + (3.0 A)(2.0 Ω) = 0 V ⇒ I1 = 5.0 A
I 2 = (3.0 A+ I1 ) = (3.0 A − 5.0 A) = 8.0 A
Finally, to determine the emf ε , we apply Kirchhoff’s loop law to the lower loop and start clockwise from the lower-
right corner of the loop:
+ε − I 2 (1.0 Ω) − (3.0 A)(2.0 Ω) = 0 V ⇒ ε − (8.0 A)(1.0 Ω) − 6.0 V = 0 V ⇒ ε = 14 V

28.53. Visualize: Please refer to Figure P28.53.


Solve: Because the 4 Ω resistor is grounded at both ends, the potential difference across this resistor is zero. That
is, no current flows through the 4 Ω resistor, and the negative terminals of both batteries are at zero potential. To
determine the current in the 2 Ω resistor, we apply Kirchhoff’s loop law. We assume that current I flows clockwise
through the 2 Ω resistor. Starting from the lower-left corner, the sum of the potential differences across various
elements in the circuit is
+ 9 V − I (2 Ω) − 3 V = 0 V ⇒ I = 3 A
28.54. Visualize: Please refer to Figure P28.54.
Solve: (a) Only bulb A is in the circuit when the switch is open. The bulb’s resistance R is in series with the internal
resistance r, giving a total resistance Req = R + r. The current is
ε 1.50 V
I bat = = = 0.231 A
R+r 6.50 Ω
This is the current leaving the battery. But all of this current flows through bulb A, so I A = I bat = 0.231 A.
(b) With the switch closed, bulbs A and B are in parallel with an equivalent resistance Req = 12 R = 3.00 Ω. Their
equivalent resistance is in series with the battery’s internal resistance, so the current flowing from the battery is
ε 1.50 V
I bat = = = 0.428 A
Req + r 3.50 Ω

But only half this current goes through bulb A, with the other half through bulb B, so I A = 12 I bat = 0.214 A.
(c) The change in I A when the switch is closed is 0.017 A. This is a decrease of 7.4%.

© Copyright 2017 Pearson Education, Inc. All rights reserved. This material is protected under all copyright laws as they currently exist.
No portion of this material may be reproduced, in any form or by any means, without permission in writing from the publisher.
Fundamentals of Circuits 28-21

28.55. Model: The battery and the connecting wires are ideal.
Visualize:

The figure shows the two circuits formed from the circuit in Figure P28.55 when the switch is open and when the
switch is closed.
Solve: (a) Using the rules of series and parallel resistors, we have simplify the circuit in two steps as shown in figure
(a) above. A battery with emf ε = 24 V is connected to an equivalent resistor of 3 Ω. The current in this circuit is
(24 V)/(3 Ω) = 8 A. Thus, the current that flows through the battery is I bat = 8 A. To determine the potential
difference Δ Vab , we will find the potentials at point a and point b and then take the difference. To do this, we need
the currents I a and I b . We note that the potential difference across the 3-Ω − 3-Ω branch is the same as the
potential difference across the 5-Ω − 1-Ω branch, so
ε = 24 V = I a (3 Ω + 3 Ω) I a = 4 A = I b
Now, Vc − I a (3 Ω) = Va , and Vc − I b (5 Ω) = Vb . Subtracting these two equations give us Δ Vab :
Va − Vb = I b (5 Ω) − I a (3 Ω) = (4 A)(5 Ω) − (4 A)(3 Ω) = +8 V
(b) Using the rules of series and parallel resistors, we simplifi the circuit as shown in figure (b), above. A battery
with emf ε = 24 V is connected to an equivalent resistor of 21 8
Ω. The current in this circuit is (24 V) 218
Ω = ( )
9.143 A. Thus, the current that flows through the battery is I bat = 9 A. When the switch is closed, points a and b are
connected by an ideal wire and must therefore be at the same potential. Thus Vab = 0 V.

28.56. Model: The connecting wires are ideal. Assume Rload = 0 Ω.


Visualize: Use the loop law and the junction law. Note that I 2 = I load .
Solve: The junction law tells us that I source = I1 + I 2 .
Now we apply Kirchhoff’s loop law to the right loop starting clockwise from the upper-right corner of the loop:
R2
− I 2 R2 + I1R1 = 0 ⇒ I1 = I2
R1
Insert this into the junction law equation.
R2 R1
I source = I 2 + I 2 ⇒ I load = I 2 = I source
R1 R1 + R2
Assess: Given a constant current source we can dictate how much of it goes to the load by adjusting the two resistors.

© Copyright 2017 Pearson Education, Inc. All rights reserved. This material is protected under all copyright laws as they currently exist.
No portion of this material may be reproduced, in any form or by any means, without permission in writing from the publisher.
28-22 Chapter 28

28.57. Model: Assume ideal connecting wires.


Visualize: Please refer to Figure P28.57. Because the ammeter we have shows a full-scale deflection with a current
of 500 μ A = 0.500 mA, we must not allow a current more than 0.500 mA to pass through the ammeter. Because we
wish to measure a maximum current of 50 mA, we must split the current in such a way that a maximum of 0.500 mA
flows through the ammeter with the remaining 49.500 mA flowing through the resistor R.
Solve: (a) The potential difference across the ammeter and the resistor is the same. Thus,
VR = Vammeter ⇒ (49.500 × 10−3 A) R = (0.500 × 10−3 A)(50.0 Ω) ⇒ R = 0.505 Ω
1 1 1
(b) The effective resistance is = + ⇒ Req = 0.500 Ω
Req 0.505 Ω 50.0 Ω

28.58. Model: The battery and the connecting wires are ideal.
Visualize:

The figure shows how to simplify the circuit in Figure P28.58 using the laws of series and parallel resistances. We
have labeled the resistors as R6 = 6 Ω, R10 = 10 Ω, R4 = 4 Ω, and R5 = 5 Ω.
Solve: R5 and R10 are combined to get R5,10 = 15 Ω, and R6 and R6 and R4 are combined to obtain R6,4 = 10 Ω.
The voltage across both branches is 24 V, so the currents through the branches are
ε 24 V ε 24 V
I 5,10 = = = 1.6 A, I 6,4 = = = 2.4 A
R5,10 15 Ω R6,4 10 Ω

Thus, the 1.6 A runs through both resistors R5 and R10 , and 2.4 A runs through resistors R6 and R4 . The potential
drop across each resistor can be found using Ohm’s law:
Δ V5 = I 5,10 R5 = (1.6 A)(5 Ω) = 8 V
Δ V10 = I 5,10 R10 = (1.6 A)(10 Ω) = 16 V
Δ V6 = I 6,4 R6 = (2.4 A)(6 Ω) = 14.4 V
Δ V4 = I 6,4 R4 = (2.4 A)(4 Ω) = 9.6 V
The table below summarizes the results:

Resistor Potential difference (V) Current (A)


5Ω 8 1.6
10 Ω 16 1.6
6Ω 14.4 2.4
4Ω 9.6 2.4

Assess: Note that the potential differences across both branches of the circuit sum to 24 V, as required.

© Copyright 2017 Pearson Education, Inc. All rights reserved. This material is protected under all copyright laws as they currently exist.
No portion of this material may be reproduced, in any form or by any means, without permission in writing from the publisher.
Fundamentals of Circuits 28-23

28.59. Model: The battery and the connecting wires are ideal.
Visualize:

The figure shows how to simplify the circuit in Figure P28.59 using the laws of series and parallel resistances. We
will reverse the procedure and “build up” the circuit using the loop law and junction law to find the current and
potential difference of each resistor.
Solve: Having found Req = 6 Ω, the current from the battery is I = (24 V)/(6 Ω) = 4 A. As we rebuild the circuit, note
that series resistors must have the same current I and that parallel resistors must have the same potential difference Δ V .

© Copyright 2017 Pearson Education, Inc. All rights reserved. This material is protected under all copyright laws as they currently exist.
No portion of this material may be reproduced, in any form or by any means, without permission in writing from the publisher.
28-24 Chapter 28

In Step 1 of the figure above, the 6 Ω equivalent resistor is returned to 2 Ω and 4 Ω resistors in series. Both resistors
must have the same 4 A as the 6 Ω equivalent resistor. We use Ohm’s law to find Δ V2 = 8 V and Δ V4 = 16 V. As a
check, 8 V + 16 V = 24 V, which was ΔV of the battery. In Step 2, the 4 Ω resistor is returned to the two 8 Ω
resistors in parallel. Both resistors must have the same Δ V = 16 V as the 4 Ω resistor. From Ohm’s law,
I8 = (16 V)/(8 Ω) = 2 A. As a check, 2 A + 2 A = 4 A, which was the current I of the 4 Ω equivalent resistor. In Step
3, the 8 Ω resistor is returned to the two 4 Ω resistors in series. Both resistors must have the same 2 A as the 8 Ω
equivalent resistor. We use Ohm’s law to find Δ V4 = 8 V. As a check, 8 V + 8 V = 16 V, which was ΔV of the 8 Ω
equivalent resistor. Finally, in Step 4, the lower 4 Ω resistor is returned to the 6 Ω and 12 Ω resistors in parallel.
Both resistors must have the same Δ V = 8 V as the 4 Ω equivalent resistor. From Ohm’s law,
I 6 = (8 V)/(6 Ω) = 4/3 A and I12 = (8 V)/(12 Ω) = 2/3 A. As a check, I 6 + I12 = 2 A, which was the current I of the
4 Ω equivalent resistor. The results are summarized in the table below.

Resistor Potential difference (V) Current (A)


2Ω 8 4
4Ω 8 2
6Ω 8 4/3
8Ω 16 2
12 Ω 8 2/3

28.60. Model: The battery and the connecting wires are ideal.
Visualize:

The figure shows how to simplify the circuit in Figure P28.60 using the laws of series and parallel resistances.
Having reduced the circuit to a single equivalent resistance, we will reverse the procedure and “build up” the circuit
using the loop law and the junction law to find the current and potential difference across each resistor.
Solve: From the last circuit in the diagram,
ε 12 V
I= = =2 A
6Ω 6Ω
Thus, the current through the battery is 2 A. As we rebuild the circuit, we note that series resistors must have the
same current I and that parallel resistors must have the same potential difference Δ V .

© Copyright 2017 Pearson Education, Inc. All rights reserved. This material is protected under all copyright laws as they currently exist.
No portion of this material may be reproduced, in any form or by any means, without permission in writing from the publisher.
Fundamentals of Circuits 28-25

In Step 1, the 6 Ω resistor is returned to a 3 Ω and 3 Ω resistor in series. Both resistors must have the same 2 A
current as the 6 Ω resistance. We then use Ohm’s law to find
Δ V3 = (2 A)(3 Ω) = 6 V
As a check, 6 V + 6 V = 12 V, which was Δ V of the 6 Ω resistor. In Step 2, one of the two 3 Ω resistances is
returned to the 4 Ω, 48 Ω, and 16 Ω resistors in parallel. The three resistors must have the same Δ V = 6 V. From
Ohm’s law,

6V 6V 1 6V 3
I4 = = 1.5 A I 48 = = A I16 = = A
4Ω 48 Ω 8 16 Ω 8
Resistor Potential difference (V) Current (A)
3Ω 6 2
4Ω 6 1.5
48 Ω 6 1/8
16 Ω 6 3/8

Assess: Note that the currents flowing through the three parallel resistors sum to the 2 A flowing through the battery,
as required.

28.61. Model: The batteries and the connecting wires are ideal.
Visualize:

The figure shows how to simplify the circuit in Figure P28.61 using the laws of series and parallel resistances.
Solve: The resistance of Req is
−1 −1
⎛ 1 1 ⎞ ⎛ 5 ⎞
Req = ⎜ + =⎜ =4 Ω
⎝ 20 Ω 5 Ω ⎟⎠ ⎟
⎝ 20 Ω ⎠
Apply Kirchhoff’s loop law, starting from the lower-right corner:
100 V
∑ Δ V = 100 V − I (2 Ω) − I (4 Ω) − I (4 Ω) ⇒ I = = 10 A
2 Ω+4 Ω+4 Ω
The voltage difference across the equivalent resistor is ΔV = IReq = (10 A)(4 Ω) = 40 V, which is also the voltage
difference across both resistors that make up this equivalent resistor. Thus, the current through the 20 Ω resistor is
ΔV 40 V
I 20 = = = 2.0 A
R20 20 Ω

© Copyright 2017 Pearson Education, Inc. All rights reserved. This material is protected under all copyright laws as they currently exist.
No portion of this material may be reproduced, in any form or by any means, without permission in writing from the publisher.
28-26 Chapter 28

28.62. Model: The batteries and the connecting wires are ideal.
Visualize:

The figure shows how to simplify the circuit in Figure P28.62 using the laws of series and parallel resistances.
Having reduced the circuit to a single equivalent resistance, we will reverse the procedure and “build up” the circuit
using the loop law and the junction law to find the current and potential difference of each resistor.
Solve: From the last circuit in the figure and from Kirchhoff’s loop law,
12 V − 3 V
I= =1 A
6 Ω+3 Ω
Thus, the current through the batteries is 1 A. As we rebuild the circuit, we note that series resistors must have the
same current I and that parallel resistors must have the same potential difference.

In Step 1 of the above figure, both resistors must have the same 1 A current. We use Ohm’s law to find
Δ V3 = (1 A)(3 Ω) = 3 V ΔV6eq = 6 V
As a check we sum the voltages around the circuit starting at the lower-left corner: 12V − 6 V − 3 V − 3 V = 0 V, as
required. In Step 2, the 6 Ω equivalent resistor is returned to the 24 Ω and 8 Ω resistors in parallel. The two resistors
must have the same potential difference Δ V = 6 V. From Ohm’s law,
6V 3 6V 1
I8eq = = A I 24 = = A
8Ω 4 24 Ω 4

© Copyright 2017 Pearson Education, Inc. All rights reserved. This material is protected under all copyright laws as they currently exist.
No portion of this material may be reproduced, in any form or by any means, without permission in writing from the publisher.
Fundamentals of Circuits 28-27

As a check, 3/4 A + 1/4 A = 1 A which was the current I of the 6 Ω equivalent resistor. In Step 3, the 8 Ω
equivalent resistor is returned to the 3 Ω and 5 Ω resistors in series, so the two resistors must have the same current
of 3/4 A. We use Ohm’s law to find
⎛3 ⎞ 9 ⎛3 ⎞ 15
Δ V3eq = ⎜ A ⎟ (3 Ω) = V ΔV5 = ⎜ A ⎟ (5 Ω) = V
⎝4 ⎠ 4 ⎝4 ⎠ 4
As a check, 9/4 V + 15/4 V = 24/4 V = 6 V, which was ΔV of the 8 Ω equivalent resistor. In Step 4, the 3 Ω
equivalent resistor is returned to 4 Ω and 12 Ω resistors in parallel, so the two must have the same potential
difference Δ V = 9/4 V. From Ohm’s law,
9/4 V 9 9/4 V 9
I4 == A I12 = = A
4Ω 16 12 Ω 48
As a check, 9/16 A + 9/48 A = 3/4 A, which was the same as the current through the 3 Ω equivalent resistor. The
results are summarized in the table below.

Resistor Potential difference (V) Current (A)


24 Ω 6 1/4
3Ω 3 1
5Ω 15/4 3/4
4Ω 9/4 9/16
12 Ω 9/4 9/48

28.63. Model: The wires and batteries are ideal.


Visualize:

Solve: Assign currents I1, I 2 , and I 3 as shown in the figure above. If I 3 turns out to be negative, we’ll know it
really flows right to left. Apply Kirchhoff’s loop rule counterclockwise to the top loop from the top-right corner:
− I1 (5 Ω) + 12 V − I 3 (10 Ω) + 3 V = 0
Apply the loop rule counterclockwise to the bottom loop starting at the lower-left corner:
− I 2 (5 Ω) + 9 V − 3 V + I 3 (10 Ω) = 0
Note that, because we went against the current direction through the 10 Ω resistor, the potential increased across this
resistor. Apply the junction rule to the right middle junction:
I1 = I 2 + I3.
These three equations can be solved for the current I 3:
9
(− I1 + I 2 )(5 Ω) + 9 V − 2 I 3 (10 Ω) = 0 ⇒ − I 3 (5 Ω) + 9 V − 2 I 3 (10 Ω) = 0 ⇒ I3 = A
25
The result is I 3 = 9/25 A = 0 .12 A flowing from left to right (as shown in the figure above).

© Copyright 2017 Pearson Education, Inc. All rights reserved. This material is protected under all copyright laws as they currently exist.
No portion of this material may be reproduced, in any form or by any means, without permission in writing from the publisher.
28-28 Chapter 28

28.64. Model: The wires and batteries are ideal.


Visualize:

Solve: If no power is dissipated in the 200 Ω resistor, the current through it must be zero. To see if this is possible, set up
Kirchhoff’s rules for the circuit, then assume the current through the 200 Ω resistor is zero and see if there is a solution.
Assume the unknown battery is oriented with its positive terminal at the top and define currents I1, I 2 , and I 3 as
shown in the figure above. Apply Kirchhoff’s loop rule clockwise to the left loop:
50 V − I1 (100) − I 3 (200) = 0
Apply Kirchhoff’s loop rule counterclockwise to the right hand loop:
ε − I 2 (300 Ω) − I 3 (200 Ω) = 0
The junction rule yields
I1 + I 2 = I3.
Now assume I 3 = 0 and solve for ε . In that case, the first equation gives
50 V 1
I1 = = A.
100 Ω 2
From the third equation, I 2 = − I1, so the second equation gives us

⎛ 1 ⎞
ε = I 2 (300 Ω) = ⎜ − A ⎟ (300 Ω) = − 150 V
⎝ 2 ⎠
Thus ε = 150 V and it is oriented with the positive terminal on the bottom, opposite to our guess.

28.65. Model: The wires are ideal, but the batteries are not.
Visualize:

Solve: (a) The good battery alone can drive a current through the starter motor
12 V
I= = 200 A
(0.01 Ω + 0.05 Ω)
(b) Alone, the dead battery drives a current
8.0 V
I= = 14.5 A ≈ 15 A
(0.50 Ω + 0.05 Ω)

© Copyright 2017 Pearson Education, Inc. All rights reserved. This material is protected under all copyright laws as they currently exist.
No portion of this material may be reproduced, in any form or by any means, without permission in writing from the publisher.
Fundamentals of Circuits 28-29

(c) Let I1, I 2 , and I 3 be defined as shown in the figure above. Kirchhoff’s laws applied to the good- and dead-
battery loop, good-battery and starter-motor loop, and the top middle junction yield three equations in the three
unknown currents:
12 V − I1 (0.01 Ω) − I 3 (0.05 Ω) = 0
12 V − I1 (0.01 Ω) − I 2 (0.50 Ω) − 8.0 V = 0
I1 = I 2 + I 3
Substituting for I1 from the third equation into the first and second equations gives
12 V − I 2 (0.01 Ω) − I 3 (0.06 Ω) = 0
4 V − I 2 (0.51 Ω) − I 3 (0.01 Ω) = 0
Solving for I 2 from the first equation,
12 V − I 3 (0.06 Ω)
I2 =
(0.01 Ω)
Substituting into the second equation and solving for I 3 yields the current through the starter motor is 199 A. To a
single significant figure, this is 200 A.
(d) Substituting the value for I 3 into the expression for I 2 yields the current through the dead battery as 3.9 A. To a
single significant figure, this is 4 A.
Assess: The good battery is charging the dead battery as well as running the started motor. A total of 203 A flows
through the good battery.

28.66. Model: The wires and batteries are ideal.


Visualize:

Solve: The circuit is redrawn above for clarity and the currents are shown. We must find I5 .
Repeatedly apply Kirchhoff’s rules to the loops. The loop rule applied clockwise about the three triangles yields
Left: 9 V − I1 (6 Ω) − I 3 (12 Ω) = 0 ⇒ I1 = 1.5 A − 2 I 3
Center: − I 4 (24 Ω) + I 3 (12 Ω) = 0 ⇒ I 4 = I 3 /2
Right: 15 V − I 2 (10 Ω) − I 4 (24 Ω) = 0 ⇒ I 2 = 1.5 A − 2.4 I 4
The junction rule applied at the bottom corners gives equations into which the results above may be substituted:
I1 = I 3 + I5 ⇒ 1.5 A − 2 I 3 = I3 + I 5 ⇒ I 5 = 1.5 A − 3I 3
I 4 = I 2 + I5 ⇒ I 4 = 1.5 A − 2.4 I 4 + I5 ⇒ I 5 = 3.4 I 4 − 1.5 A
Using I 4 = I 3 /2 and solving for I 3 gives
30
1.5 A − 3I3 = 3.4( I 3 /2) − 1.5 A ⇒ I3 = A
47
⎛ 30 ⎞ 201
I 5 = 1.5 A − 3 ⎜ A⎟ = − A = −0.41 A
⎝ 47 ⎠ 94
Since the result is negative, 0.40 A flows from left to right through the bottom wire.

© Copyright 2017 Pearson Education, Inc. All rights reserved. This material is protected under all copyright laws as they currently exist.
No portion of this material may be reproduced, in any form or by any means, without permission in writing from the publisher.
28-30 Chapter 28

28.67. Model: Assume ideal wires. The capacitor discharges through the resistor.
Solve: (a) The capacitor discharges through the resistor R as Q = Q0e−t /τ . For Q = Q0 /2,
Q0 ⎛1⎞ t
= Q0e −t /10 ms ⇒ ln ⎜ ⎟ = − ⇒ t = −(0.010 s)ln(0.5) = 6.9 ms
2 ⎝2⎠ 0.010 s
(b) If the initial capacitor energy is U 0 , we want the time when the capacitor’s energy will be U = U 0 /2. Noting that
U 0 = Q02 /(2 C ), this means Q = Q0 / 2. Applying the equation for the discharging capacitor gives
Q0 ⎛ 1 ⎞ t ⎛ 1 ⎞
= Q0e−t /10 ms ⇒ ln ⎜ ⎟=− ⇒ t = −(0.010 s)ln ⎜ ⎟ = 3.5 ms
2 ⎝ 2 ⎠ 0.010 s ⎝ 2⎠

28.68. Model: The capacitor discharges through the resistor R as I = I 0e−t /τ , where τ = RC. Assume negligible
resistance in the connecting leads.
Solve: (a) Taking natural logs of the current equation gives
ln( I ) = ln( I 0 ) − t / τ
Therefore, if we plot ln(I ) versus t, we should get a straight line with slope s = − 1/ τ and y-intercept equal to ln I 0 .
The slope s will be related to the resistance according to s = −1/ τ and the initial current can be found from the y-
intercept (from which we shall find the initial capacitor voltage). The plot is shown below.

The slope is s = − 0.7698, so the resistance is


1 1 1
− 0.7698 s −1 = − =− ⇒ R= = 64,952 Ω = 65 k Ω
τ RC (0.7698 s −1 )(20 μ F)
(b) The y-intercept of the graph is −6.614, so the initial current is
ln( I 0 ) = − 6.614 ⇒ I 0 = e−6.614 = 1.341 mA
Because the capacitor and the resistor are in parallel, the capacitor voltage equals the resistor voltage. Using Ohm’s
law to find the initial resistor voltage will therefore also give us the capacitor voltage:
VC = V0 = I 0 R = (1.341 mA)(64.952 kΩ) = 87 V

28.69. Model: The capacitor discharges through the resistor (the patient’s chest) according to Q = Q0e−t /τ (see
Equation 28.29), where τ = RC. Assume negligible resistance in the connecting leads and that all the resistance is
due to the patient’s chest.
Solve: At time t = 40 ms, the capacitor has lost 95% of its charge, so it has only 5% left. Therefore Q = 0.050 Q0 at that time:
Q −t −(40 ms)
= 0.050 = e−t /τ = e−t /(RC ) ⇒ R= = = 89 Ω
Q0 C ln(0.050) (150 μ F)ln(0.050)

© Copyright 2017 Pearson Education, Inc. All rights reserved. This material is protected under all copyright laws as they currently exist.
No portion of this material may be reproduced, in any form or by any means, without permission in writing from the publisher.
Fundamentals of Circuits 28-31

28.70. Model: The capacitor discharges through the resistor, and the wires are ideal.
Solve: In an RC circuit, the charge at a given time is related to the original charge as Q = Q0e−t /τ . For a capacitor
Q = C Δ V , so ΔV = Δ V0e−t /τ . From the Figure P28.70, we note that Δ V0 = 30 V and Δ V = 10 V at t = 4 ms. So,
−6 ⎛ 10 V ⎞ 4 × 10−3 s 4 × 10−3 s
10 V = (30 V)e−4 ms/R (50 × 10 F)
⇒ ln ⎜ ⎟=− ⇒ R=− = 73 Ω
⎝ 30 V ⎠ R(50 × 10−6 F) (50 × 10−6 F)ln ( 13 )
28.71. Model: The capacitor discharges through the resistors. The wires are ideal.
Solve: The charge Q on the capacitor charged to 50 V is
Q = C Δ V = (0.25 μ F)(50 V) = 12.5 μ C
When this fully charged capacitor is connected in series with a 25 Ω resistor and a 100 Ω resistor, it will dissipate
all its stored energy
Q 2 (12.5 μ C) 2
UC = = = 312.5 × 10−6 J
2C 2(0.25 μ F)
through the two resistors. Energy dissipated by the resistor is I 2 R, which means the 100 Ω resistor will dissipate
four times more energy than the 25 Ω resistor at any given time. Thus, the energy dissipated by the 25 Ω resistor is
⎛ 25 Ω ⎞
312.5 × 10−6 ⎜ −6
⎟ = 6 2.5 × 10 J = 63 μ J
⎝ 25 Ω + 100 Ω ⎠

28.72. Model: The connecting wires are ideal.


Visualize: The time constant for an RC circuit is τ = RC .
Solve: The junction law tells us that I source = I1 + I 2 .
2 2
τ series R C R R1 + R2 R + R2 ( R1 + R2 ) (15 kΩ + 25 kΩ ) = 4.3
= series = series = = 1 = =
τ parallel RparallelC Rparallel ⎛ 1 1 ⎞
−1 R 1R2 R R
1 2 (15 kΩ )( 25 kΩ )
⎜ + ⎟ R1 + R2
⎝ R1 R2 ⎠
Assess: This ratio is independent of the value of the capacitance.

28.73. Model: The battery and the connecting wires are ideal.
Visualize: Please refer to Figure P28.73.
Solve: (a) A very long time after the switch has closed, the potential difference ΔVC across the capacitor is ε . This
is because the capacitor charges until Δ VC = ε while the charging current approaches zero.
(b) The full charge of the capacitor is Qmax = C (Δ VC ) max = Cε .
(c) In this circuit, I = + dQ / dt because the capacitor is charging; that is, because the charge on the capacitor is
increasing.
(d) From Equation 28.34, capacitor charge at time t is Q = Qmax (1 − e −t /τ ). Therefore,
dQ d ⎛1⎞ ⎛ 1 ⎞ −t /τ ε −t /τ
I= = Cε (1 − e−t /τ ) = Cε ⎜ ⎟ e−t /τ = Cε ⎜ ⎟e = e
dt dt ⎝τ ⎠ ⎝ RC ⎠ R

© Copyright 2017 Pearson Education, Inc. All rights reserved. This material is protected under all copyright laws as they currently exist.
No portion of this material may be reproduced, in any form or by any means, without permission in writing from the publisher.
28-32 Chapter 28

A graph of I as a function of t is shown below.

28.74. Model: The connecting wires are ideal. The capacitors discharge through the resistors.
Visualize:

The figure shows how to simplify the circuit in Figure P28.74 using the laws of series and parallel resistors and the
laws of series and parallel capacitors.
Solve: The 30 Ω and 20 Ω resistors are in parallel and are equivalent to a 12 Ω resistor. This 12 Ω resistor is in
series with the 8 Ω resistor so the equivalent resistance of the circuit Req = 20 Ω. The two 60 μ F capacitors are in
series producing an equivalent capacitance of 30 μ F. This 30 μ F capacitor is in parallel with the 20 μ F capacitor
so the equivalent capacitance Ceq of the circuit is 50 μ F. The time constant of this circuit is

τ = ReqCeq = (20 Ω)(50 μ F) = 1.0 ms


The current due to the three capacitors through the 20 Ω equivalent resistor is the same as through the 8 Ω resistor.
So, the voltage across the 8 Ω resistor follows the decay equation V = V0e −t /τ . For V = V0 /2, we get
V0 ⎛1⎞ t
= V0e−t /1.0 ms ⇒ ln ⎜ ⎟ = − ⇒ t = 0.69 ms
2 2
⎝ ⎠ 1.0 ms

28.75. Model: The connecting wires are ideal.


Visualize: The time constant for an RC circuit is τ = RC .
Solve: (a) We simply compute how long two time constants is.
2τ = 2 RC = 2 ( 5.0 Ω )(120 μ F ) = 1.2 ms
(b) Find the rate energy is dissipating (i.e., the power) at t = 250 μ s.
(Δ V ) 2 ( ΔV0 e−t /τ ) 2 ((220 V) e−250 μs/0.60 ms ) 2
P= = = = 4.2 kW
R R 5.0 Ω

© Copyright 2017 Pearson Education, Inc. All rights reserved. This material is protected under all copyright laws as they currently exist.
No portion of this material may be reproduced, in any form or by any means, without permission in writing from the publisher.
Fundamentals of Circuits 28-33

(c) The total energy dissipated is just the energy stored in the capacitor originally.
U C = 12 C (ΔV ) 2 = 12 (120 μ F)(220 V) 2 = 2.9 J

Assess: The answer to (b) can also be obtained by taking the time derivative of U C = 12 C (Δ V ) 2 .

28.76. Model: The connecting wires are ideal.


Visualize: The time constant for an RC circuit is τ = RC .
Solve: The current in a discharging RC circuit is I = I 0e−t /τ . The current will be large (above the danger level) at
first and then decay away to safe levels. We compute the time it takes to reach 50 mA.
ΔV0 −t /τ IR t
I= e ⇒ ln =− ⇒
R Δ V0 τ
IR (50 mA)(1.8 k Ω)
t = −τ ln = −(1.8 k Ω)(120 μ F)ln = 0.11 s
ΔV0 150 V
Assess: The lesson is to discharge capacitors with a screwdriver with an insulated handle before poking around in
electronic equipment.

28.77. Model: T = TH + TL .
Visualize: We are given f = 10 MHz ⇒ T = 0.10 μ s = 100 ns and C = 500 pF.
Solve: Given T = 100 ns we surmise that TH = 75 ns and TL = 25 ns. Use TL to solve for R2.
TL 25 ns
TL = R2C ln 2 ⇒ R2 = = = 72 Ω
C ln 2 (500 pF)ln 2
Now solve the TH equation for R1.
TH 75 ns
TH = ( R1 + R2 )C ln 2 ⇒ R1 = − R2 = − 72 Ω = 144 Ω
C ln 2 (500 pF)ln 2
TH − TL
Note that R1 = and R1 = 2 R2 . We report the last number to two significant figures: R1 = 140 Ω.
C ln 2
Assess: This capacitor has a small capacitance, but these are easy to find. The resistances are typical.

Challenge Problems

28.78. Model: The wires and batteries are ideal.


Visualize:

Solve: The circuit has been redrawn for clarity. Assign the currents I1, I 2 , and I 3 as shown in the figure. To find the
power dissipated by the 2 Ω resistor, we must find the current through it. Apply Kirchhoff’s loop rule clockwise to
the left loop from the bottom-left corner:
+ 12 V − I1 (4 Ω) − I 3 (2 Ω) = 0.

© Copyright 2017 Pearson Education, Inc. All rights reserved. This material is protected under all copyright laws as they currently exist.
No portion of this material may be reproduced, in any form or by any means, without permission in writing from the publisher.
28-34 Chapter 28

Apply the loop rule clockwise to the right loop starting at the top-right corner:
+ 15 V − I 2 (4 Ω) + I 3 (2 Ω) = 0
Note that, because we went against the current direction through the 2 Ω resistor, the potential increased. Apply the
junction rule to the lower junction:
I1 = I 2 + I 3
These three equations can be solved for the current I 3 by subtracting the second equation from the first, then making the
substitution I 2 − I1 = − I3 that was derived from the third equation. The result is I 3 = − 3/8 A. The power dissipated in
the resistor is
2
⎛ 3 ⎞ 9
P2 Ω = I 32 R2 = ⎜ − A ⎟ (2 Ω) = W = 0.3 W
⎝ 8 ⎠ 32

28.79. Solve: The resistivity of aluminum is 2.8 × 10−8 Ω m and we want the wire to dissipate 7.5 W when connected
to a 1.5 V battery. The resistance of the wire must be
V2 V 2 (1.5 V)2
P= ⇒ R= = = 0.30 Ω
R P 7.5 W
Using the formula for the resistance of a wire,
L L
R=ρ ⇒ 0.30 Ω = (2.8 × 10−8 Ω m) 2 ⇒ L = (3.366 × 107 m −1 )r 2
A πr
We need another relation connecting L and r. Making use of the mass density of aluminum gives
1.0 × 10−3 kg
= 2700 kg/m3 ⇒ r 2 L = 1.179 × 10−7 m3
π r 2L
Using the value of L obtained above,
r 2 (3.366 × 107 m −1 )r 2 = 1.179 × 10−7 m3 ⇒ r 4 = 3.50 × 10−15 m 4 ⇒ r = 2.43 × 10−4 m = 0.243 mm
Thus, the diameter of the wire is 0.49 mm and the length is
L = (3.366 × 107 m −1 )(2.43 × 10−4 m) 2 = 2.0 m
Assess: It is reasonable to make a 2.0 m long wire with a diameter of 0.49 mm from an aluminum block of 1.0 g.

28.80. Model: Assume the battery and the connecting wires are ideal.
Visualize: Please refer to Figure CP28.80.
Solve: (a) If the switch has been closed for a long time, the capacitor is fully charged and there is no current flowing
through the right branch that contains the capacitor. Therefore, a voltage of 60 V appears across the 60 Ω resistor
and a voltage of 40 V appears across the 40 Ω resistor. That is, maximum voltage across the capacitor is 40 V.
Thus, the charge on the capacitor is
Q0 = ε C = (40 V)(2.0 × 10−6 F) = 80 μ C
(b) Once the switch is opened, the battery is disconnected from the capacitor. The capacitor C has two resistances
(10 Ω and 40 Ω, which give a 50 Ω equivalent resistance) in series and discharges according to Q = Q0e −t /RC .
For Q = 0.10 Q0 ,
t
0.10Q0 = Q0e −t /[(50 Ω )(2.0 μ F)] ⇒ ln(0.10) = −
(50 Ω)(2.0 μ F)
t = −(50 Ω)(2.0 μ F)ln(0.10) = 0.23 ms

© Copyright 2017 Pearson Education, Inc. All rights reserved. This material is protected under all copyright laws as they currently exist.
No portion of this material may be reproduced, in any form or by any means, without permission in writing from the publisher.
Fundamentals of Circuits 28-35

28.81. Model: The battery and the connecting wires are ideal.
Visualize: Please refer to Figure 28.28a.
Solve: After the switch closes at t = 0 s, the capacitor begins to charge. At time t, let the current and the charge in
the circuit be i and q, respectively. Also, assume clockwise direction for the current i. Using Kirchhoff’s loop law and
starting clockwise from the lower-left corner of the loop,
q dq q dq dt
+ε − iR − = 0 ⇒ ε = R+ ⇒ RCdq = (ε C − q )dt ⇒ =
C dt C ε C − q RC
Integrating both sides gives
Q dq dt Q t t
=t ⇒ − ⎡⎣ln(ε C − q ) ⎤⎦0 = ⇒ − ln(ε C − Q ) + ln(ε C ) =
0 εC − q 0 RC RC RC
⎛ εC − Q ⎞ t ε C − Q −t /RC
ln ⎜ ⎟ = − RC ⇒ =e ⇒ Q = ε C (1 − e−t /RC )
⎝ εC ⎠ εC
Letting Qmax = ε C and τ = RC , we get Q = Qmax (1 − e −t /τ ).

28.82. Model: The battery and the connecting wires are ideal.
Visualize: Please refer to Figure 28.28a.
Solve: (a) According to Equation 28.34, the charge on the capacitor increases according to Q = Qmax (1 − e −t /τ )
during charging. Therefore, the current in the circuit behaves as
dQ ⎛ 1⎞ ε C −t /RC ε −t /RC
I= = Qmax ⎜ − ⎟ (−e −t /τ ) = e = e
dt ⎝ τ⎠ RC R
Using Equation 28.8, the power supplied by the battery as the capacitor is being charged is
⎛ε ⎞ ε 2 −t /RC
Pbat = I ε = ⎜ e−t /RC ⎟ ε = e
⎝R ⎠ R
Because Pbat = dU / dt , we have
2
∞ε ε2 ⎡ ∞
dU = Pbat dt ⇒ ∞
∫ dU = 0 Pbat dt = 0 e−t /RC dt = − RCe−t /RC ⎤ = ε 2C
R R ⎣ ⎦0
That is, the total energy that has been supplied by the battery when the capacitor is fully charged is ε 2C.
(b) The power dissipated by the resistor as the capacitor is being charged is
ε2
Presistor = I 2 R = Re −2t /RC
R2
Because Presistor = dU / dt , we have

∞ ε2 ε 2 ⎡ RC ⎤ ε 2C
dU = Presistor dt ⇒ ∫ dU = e−2t /RC dt ⇒ U resistor = ⎢ − e−2t /RC ⎥ =
0 R R ⎣ 2 2
⎦0
(c) The energy stored in the capacitor when it is fully charged is
2
1 Qmax 1 C 2ε 2 1 2
UC = = = Cε
2 C 2 C 2
(d) For energy conservation, the energy delivered by battery is equal to the energy dissipated by the resistor R plus
the energy stored in the capacitor C. This is indeed the case because U bat = ε 2C , U resistor = 12 ε 2C , and U C = 12 ε 2C.

© Copyright 2017 Pearson Education, Inc. All rights reserved. This material is protected under all copyright laws as they currently exist.
No portion of this material may be reproduced, in any form or by any means, without permission in writing from the publisher.
28-36 Chapter 28

28.83. Model: The battery and the connecting wires are ideal.
Visualize: Please refer to Figure CP28.83.
Solve: (a) During charging, when the neon gas behaves like an insulator, the charge on the capacitor increases
according to Equation 28.34; that is, Q = Qmax (1 − e −t /τ ). Because Q = C ΔVC ,

Δ VC = Δ VC0 (1 − e−t /τ ) = ε (1 − e−t /τ )


Let us say that the period of oscillation begins when Δ V = Voff and it ends when Δ V = Von . Then,
ε − Voff toff ⎛ ε ⎞
Voff = ε (1 − e−toff /τ ) ⇒ = e−toff /τ ⇒ = ln ⎜ ⎟
ε τ ⎝ ε − Voff ⎠
Because the period T = ton − toff , we have
⎛ ε ⎞ ⎛ ε ⎞ ⎛ ε − Voff ⎞ ⎛ ε − Voff ⎞
T = τ ln ⎜ ⎟ − τ ln ⎜ ⎟ = τ ln ⎜ ⎟ = RC ln ⎜ ⎟
⎝ ε − Von ⎠ ⎝ ε − Voff ⎠ ⎝ ε − Von ⎠ ⎝ ε − Von ⎠
(b) Substituting the given values into the above expression and noting that and T = 1/f = 0.10 s gives
⎛ 90 V − 20 V ⎞ 0.10 s
0.10 s = R (10 × 10−6 F)ln ⎜ ⎟ ⇒ R= = 5.1 k Ω
⎝ 90 V − 80 V ⎠ (10 × 10−6 F)ln(7)

© Copyright 2017 Pearson Education, Inc. All rights reserved. This material is protected under all copyright laws as they currently exist.
No portion of this material may be reproduced, in any form or by any means, without permission in writing from the publisher.

You might also like